You are on page 1of 28

September 2006

Diagnosis And Management Of Authors


Volume 8, Number 9

North American Snake And Melissa W Costello, MD


Assistant Professor of Emergency Medicine,
University of South Alabama, Mobile, Alabama
Scorpion Envenomations Alan Heins, MD
Assistant Professor of Emergency Medicine,
University of South Alabama, Mobile, Alabama
Case #1: A 45-year-old man presents, via rural EMS, with a chief complaint
Daniel A Zirkin, MD
of “snakebite.” EMS reports that the patient ran over a 6 to 8ft snake with his Resident Physician, Department of Emergency
pickup truck, cutting it in half. The patient doubled back to pick up the dead Medicine, Emory University, Atlanta, Georgia
snake (foreseeing a nice new pair of snakeskin boots) and when he grabbed it, Peer Reviewers
the snake “came around” and bit his hand. He has two puncture wounds almost Frank Lovecchio, DO, MPH, FACEP
Medical Director, Banner Good Samaritan Regional
7cm apart on his right hand surrounded by dark purple ecchymosis and the start Poison Center; Research Director, Maricopa Medical
of bullae formation. He has edema and erythema of his hand and arm to the Center, Department of EM; Associate Professor, AZ
elbow and is having fasciculations of most of his large skeletal muscle groups. He College of Osteopathic Medicine
is in pain and EMS reports that he seemed to be getting progressively confused Robert Barish, MD
Professor of Emergency Medicine, Vice Dean for
and lethargic during their 70 minute transport. He is hypotensive, tachycardic,
Clinical Affairs, University of Maryland School of
and is oozing blood from his IV sites. You realize that his life depends on your Medicine, Baltimore, MD
actions... Charles Stewart, MD, FAAEM, FACEP
Case #2: A 30-year-old man presents with a chief complaint of “snakebite.” Emergency Physician, Colorado Springs, CO
About 30 minutes prior to arrival he was wading in murky water cleaning debris CME Objectives
out of a stream and felt a bite on his hand. As he pulled his hand out of the water, Upon completion of this article, you should be able to:
he briefly caught a glimpse of a snake as it swam away; he gives a fairly good 1. Name the types of venomous snakes native to
the United States.
description of a small copperhead. He has two small punctures on his right index 2. Describe the key characteristics for identifying
finger about 1 cm apart. He is currently pain free, without erythema, swelling, or venomous versus non-venomous snakes.
ecchymosis. His vitals and remaining physical exam are normal. He wants to 3. Explain the grading system for envenomations.
4. Describe the indications and administration of
know if he can go home... CroFabTM.
Case #3: A 39-year-old man presents to your Florida ED about four hours 5. Effectively evaluate and manage severe snake
after being bitten on the forearm by a small red, yellow, and black snake while and scorpion envenomations.
6. Prevent and treat adverse effects of antivenom
clearing brush. The snake escaped and was thought to be a king snake. There was administration.
minimal pain at the site of the bite and no swelling or bleeding, so he continued 7. Describe multiple resources available to assist in
working. The patient started to get worried when he developed twitching in his the management and treatment of snake enveno-
mations.
arms and face about one hour before arrival at the ED. Soon after the twitching
Date of original release: September 12, 2006.
started, he began having difficulty talking and swallowing and had an episode of Date of most recent review: September 1, 2006.
double vision, prompting him to call 911. On arrival, the patient is awake, alert, See “Physician CME Information” on back page.

Editor-in-Chief Health Science Center, New Orleans, HSC/Jacksonville, FL. Alfred Sacchetti, MD, FACEP, Beth Wicklund, MD, Regions Hospital
LA. Assistant Clinical Professor, Emergency Medicine Residency,
Andy Jagoda, MD, FACEP, Professor Gregory L Henry, MD, FACEP, CEO,
Wyatt W Decker, MD, Chair and Department of Emergency Medicine, EMRA Representative.
and Vice-Chair of Academic Affairs, Medical Practice Risk Assessment,
Associate Professor of Emergency Thomas Jefferson University,
Inc; Clinical Professor of Emergency
Department of Emergency Medicine; Medicine, Mayo Clinic College of
Medicine, University of Michigan, Ann
Philadelphia, PA. International Editors
Mount Sinai School of Medicine; Medicine, Rochester, MN.
Medical Director, Mount Sinai Hospital, Arbor. Corey M Slovis, MD, FACP, FACEP, Valerio Gai, MD, Senior Editor,
New York, NY. Francis M Fesmire, MD, FACEP, Professor and Chair, Department of Professor and Chair, Dept of EM,
Keith A Marill, MD, Instructor,
Director, Heart-Stroke Center, Emergency Medicine, Vanderbilt University of Turin, Italy.
Department of Emergency Medicine,
Erlanger Medical Center; Assistant University Medical Center, Nashville,
Associate Editor Massachusetts General Hospital,
TN.
Peter Cameron, MD, Chair, Emergency
Professor, UT College of Medicine,
Harvard Medical School, Boston, MA. Medicine, Monash University; Alfred
John M Howell, MD, FACEP, Clinical Chattanooga, TN.
Jenny Walker, MD, MPH, MSW, Hospital, Melbourne, Australia.
Professor of Emergency Medicine, Charles V Pollack, Jr, MA, MD, FACEP,
Michael J Gerardi, MD, FAAP, FACEP, Assistant Professor; Division Chief,
George Washington University, Professor and Chair, Department of Amin Antoine Kazzi, MD, FAAEM,
Director, Pediatric Emergency Family Medicine, Department of
Washington, DC; Director of Academic Emergency Medicine, Pennsylvania Associate Professor and Vice Chair,
Medicine, Children’s Medical Center, Community and Preventive Medicine,
Affairs, Best Practices, Inc, Inova Hospital, University of Pennsylvania Department of Emergency Medicine,
Atlantic Health System; Department of Mount Sinai Medical Center, New
Fairfax Hospital, Falls Church, VA. Health System, Philadelphia, PA. University of California, Irvine;
Emergency Medicine, Morristown York, NY.
American University, Beirut, Lebanon.
Memorial Hospital, NJ. Michael S Radeos, MD, MPH,
Editorial Board Assistant Professor of Emergency
Ron M Walls, MD, Professor and Chair,
Hugo Peralta, MD, Chair of Emergency
Michael A Gibbs, MD, FACEP, Chief, Department of Emergency Medicine,
William J Brady, MD, Associate Medicine, Lincoln Health Center, Services, Hospital Italiano, Buenos
Department of Emergency Medicine, Brigham & Women’s Hospital, Boston,
Professor and Vice Chair, Department Bronx, NY. Aires, Argentina.
Maine Medical Center, Portland, ME. MA.
of Emergency Medicine, University of Robert L Rogers, MD, FAAEM, Maarten Simons, MD, PhD,
Steven A Godwin, MD, FACEP,
Virginia, Charlottesville, VA. Assistant Professor and Residency Research Editors Emergency Medicine Residency
Assistant Professor and Emergency
Peter DeBlieux, MD, LSUHSC Director, Combined EM/IM Program, Director, OLVG Hospital, Amsterdam,
Medicine Residency Director, Nicholas Genes, MD, Mount Sinai
Professor of Clinical Medicine; LSU University of Florida University of Maryland, Baltimore, The Netherlands.
Emergency Medicine Residency.
MD.
and anxious. His heart rate is 92, blood pressure 140/85, tem- snakebites and scorpion stings is unclear, supported
perature 98.2°F orally, respiratory rate 22, and oxygen satura- by large, but not population-based studies. The
tion 95% on room air. You note right eye ptosis; the pupils are
American Association of Poison Control Centers
equal, round, and reactive to light, but the right eye does not
(AAPCC) compiles data from their Toxic Exposure
move past the midline on lateral gaze. Mild dysarthria is pres-
ent. There are two tiny puncture marks on the right forearm Surveillance System (TESS) and publishes a compre-
with 1-2cm of surrounding ecchymosis, but no swelling. hensive annual analysis of all toxic exposures,
During your exam, respirations became shallower and labored, including envenomations, reported to all of the US
with some snoring upper airway sounds. You realize that this Poison Control Centers. These reports include infor-
might be more than just the bite of a king snake...
mation on exposures, ED use, and clinical outcomes
including death.1-2 In spite of this centralized clear-

S nakes, both native and imported, and scorpions


produce clinically important envenomations in
the United States. While envenomations have long
inghouse/reporting system, the consensus among
experts in the field is that relying on self-reported
data in the cases of envenomations that do not
been considered the purview of the Southeastern US require ED visits results in a true incidence on
and the desert Southwest, the widespread and ever- envenomation that is as high as three to four times
growing population of people who maintain these the number reported by TESS.3-5
animals in collections and as “pets” has made the The body of current literature on North
presentation of a snakebite or scorpion sting possible American snake envenomations can generally be
in any emergency department (ED) in the country. divided into two categories; pre-CroFabTM and post-
These animals can produce devastating injuries and CroFabTM. The Crotalidae Polyvalent Immune Fab
must be recognized promptly and treated appropri- Ovine (CroFabTM) was approved by the US Food and
ately to prevent significant morbidity and death. Drug Administration in October of 2000 essentially
Not all envenomations need treatment. Indeed, there replacing the older Antivenom Crotalidae Polyvalent
are times when the risk of treatment outweighs the (ACP) available since 1954. Indications and adminis-
benefit. However, failure to aggressively treat in the tration of CroFabTM will be discussed in detail later in
appropriate situation may result in a disastrous out- this article. Dozens of articles have been published
come for the patient and for the clinician who does on the topic of snake envenomations since the
not meet the standard of care expected of the emer- approval of CroFabTM, but the vast majority are
gency medicine specialist. review articles by a small pool of authors. Very little
This article will divide the discussion of snakes original research has been done since the CroFabTM
and the management of snake and scorpion enveno- research in 1999. The research that has been done
mations into two sections. The first section will deal has typically been in small populations and does not
exclusively with pit vipers (Crotalids), their identifi- include a single randomized, blinded clinical trial of
cation, envenomation signs and symptoms, and any medication, device, or technique for snakebite
treatment with CroFabTM. Changes and controversies management.
in the care of patients with Crotalid envenomations The most recent comprehensive review identi-
will also be addressed. The second section will deal fied was “North American snake envenomation:
with coral snakes and the identification and treat- diagnosis, treatment and management” by Gold,
ment issues that they present, along with a brief dis- Barish, and Dart in the Emergency Medicine Clinics of
cussion of scorpion stings and the “exotics:” The North America.4 This group of authors has written
non-native captive snakes that are found more and extensively in the emergency medicine and wilder-
more commonly in the US, both in zoos and as pets. ness medicine literature on the topic of snake enven-
omations. In addition to the review article above,
Critical Appraisal Of The Literature
they participated in a four article symposium in the
Unfortunately, evidence from well-conducted clinical Annals of Emergency Medicine6 (presenting the data
research is in short supply to guide the management from the October 1999, North American Congress of
of envenomations. The literature supporting the ED Clinical Toxicology’s, “Advances in the Management
evaluation and treatment of envenomations is gener- of Snakebite” Symposium), and a full review of ven-
ally weak, with few randomized trials or meta-analy- omous snakebites in the New England Journal of
ses, and many review articles rehashing the same Medicine.5 Most of the recent research-based litera-
weak literature. Even the epidemiology of ture has focused on the limited areas of field man-

Emergency Medicine Practice© 2 September 2006 • EBMedice.net


agement changes, trials of venom extraction devices, Epidemiology And Etiology
surgical management, and the expansion of the use
Given the multitude of people who have an intense
of CroFabTM to non-Crotalidae envenomations; all of
and occasionally irrational fear of snakes, it is hard
which will be discussed in this article.
to believe that snakebites are a relatively infrequent
Coral snake envenomations are not treated with
occurrence. Even harder to believe, is that fatalities
CroFabTM and thus will be reviewed separately. It is
from snake envenomations are exceedingly rare. It is
fairly rare and human treatment studies include only
estimated that there are approximately 45,000
case series.7 Additionally, one well-designed animal
snakebites per year in the United States.12 Seven
model study explored a technique for first aid of
thousand to 8000 of these bites are attributed to ven-
coral snake envenomations.8 Similarly, the evalua-
omous snakes, but these bites only result in five to
tion and management of scorpion envenomations is
six deaths annually.3,12 The Toxic Exposure
supported by only one randomized, placebo-con-
Surveillance System (TESS) report for 2004 breaks
trolled trial from Tunisia that enrolled a group with
down snakebites by species, with 98% of bites by
few severely affected people, and a systematic
venomous snakes in the US from the Crotalids; there
review including a randomized trial and three cohort
were a total of 5046 reported bites, with 167 major
studies.9,10
complications and two deaths. TESS reported 97
Non-native venomous snakes represent a hetero-
coral snake envenomations in 2004, with four major
geneous group of animals kept by collectors and
complications and no deaths. Envenomations by poi-
zoos. These snakes cause some dangerous enveno-
sonous exotic snakes resulted in 131 reported cases
mations each year in the United States, but the
in 2004. These bites were more severe than coral
majority of the literature on evaluation and treat-
snakebites with morbidity and mortality rates similar
ment of these snakes is published in the countries
to that of rattlesnakes; about 35% moderate injury,
where the snakes are native. Most of these reports
7% major injury, and one death.
are low quality, but a few randomized, controlled tri-
One of the major limitations of the data reported
als support practice and are available through online
by TESS is that, generally speaking, the data is com-
databases by searching the specific species of snake
piled only from snake envenomations that result in
responsible for the envenomation.
visits to a physician (ED or other) or a call to the
Clinical guidelines are of virtually no help in
regional or national Poison Control Center. Many
guiding the assessment and treatment of envenoma-
people who sustain minor envenomations or dry
tions. A search of the National Guidelines
bites (bites where no venom is injected) will not seek
Clearinghouse using various key words, including
medical care. It is reasonable to conclude that the
snakebite, envenomation, rattle snake, coral snake,
patients who do not seek medical care generally will
scorpion, cobra, krait, venom, etc. yielded only one
not take the time to report their envenomation to
relevant guideline, entitled, “First aid: 2005
TESS. Thus, authors of scientific and medical writ-
International Consensus Conference on
ings are left to speculate on the true incidence of
Cardiopulmonary Resuscitation and Emergency
envenomations. The current consensus seems to be
Cardiovascular Care Science with Treatment
based on the estimate given by Parish in a 1954
Recommendations,” published in Circulation.11 The
report on the incidence of treated snakebites in the
two suggestions for the first aid of snakebites are:
US (45,000),12 and a report by Langley and Morrow
1) First aid providers should not apply suction to
estimating that the actual incidence and death rate
snakebite envenomation sites; this recommendation
from snakebites was three to four times that reported
is supported by a few Class II and III studies.
by TESS.3 The majority of deaths occur among chil-
2) Properly performed pressure immobilization
dren and the elderly, among those for whom
is recommended for first aid treatment of Elapid
antivenom is not given, is postponed, or is adminis-
snakebites. The first aid provider creates this
tered in insufficient quantities, and among members
pressure by applying a snug bandage that allows
of fundamentalist religious groups who handle poi-
a finger to slip under the bandage; this recommen-
sonous snakes during religious rituals.13
dation is supported by a single Class III study in
Rattlesnakes, with their presence in virtually all of
a porcine model of a coral snake envenomation.
the continental US and the high potency of their
venom, are responsible for the majority of snakebite

EBMedicine.net • September 2006 3 Emergency Medicine Practice©


fatalities with the diamondback rattlesnake account- sent any of the venomous species found in the
ing for 95% of these fatal bites.14-15 world, but are most commonly cobras, kraits, and
Snakebites are more common during the spring true vipers (as opposed to the pit vipers).
and summer months when both snakes and people
are more active outdoors. Very little has changed in Scorpions
the literature regarding the demographics of snake Scorpions are arthropods in the class Arachnida,
envenomations since 1966, when Parish published sharing some characteristics with spiders, and are
one of the first comprehensive survey results in found worldwide. Most species of scorpions deliver
Public Health Reports.12 Best summarized by Gold et venom that is not dangerous to humans, but there
al, “the majority of victims remain men between the are a noteworthy few that deliver a potent, although
ages of 17 and 27 years. More than 95% of the bites not deadly, venom. In the US, scorpions are found
are on the extremities, and most occur between April primarily in the desert Southwest, with only one
and October, the peak months being in July and species, the Bark scorpion Centruroides sculpturatus
August.”4 Nonetheless, an emergency physician in (aka exilicauda), dangerous to humans. The Bark
any part of the country can potentially see snakebites scorpion is only found in Arizona and Northern
at any time of the year, often from deliberate expo- Mexico.
sure to captive snakes.16 The South African and Tunisian scorpions are
Of the nearly 3000 species of venomous and non- very different from the Southwestern scorpion. In
venomous snakes worldwide, there are less than 40 the US, most envenomations are treated at home,
species of venomous snakes in North America. To with very few advancing to pulmonary edema or
simplify things even further for the non-herpetolo- shock. Most of the large case series and controlled
gist, the venomous species of North America can be trials have been performed outside the US, and cau-
divided into four main types: Rattlesnakes (consist- tion must be used when extrapolating these studies
ing of species from both the Crotalus and the to the management of patients in the US.
Sistrurus genus), copperheads (genus Agkistrodon),
cottonmouths (genus Agkistrodon), and coral snakes. Pathophysiology
The first three types are members of the Viper family
Pit Vipers/Crotalids
and the Crotalinae subfamily; coral snakes are North
Crotalid venom is a stunningly complex mixture of
America’s only native member of the family of
proteins that include proteolytic enzymes, collage-
snakes called Elapids.4,17 The family Elapidae also
nases, phospholipases, nucleotidase, hyaluronidase,
includes many of the most dangerous snakes in the
acetylcholinesterase, and amino acid oxidase. In
world including cobras, kraits, adders, and mambas.
addition, it contains elemental metals, amino acids,
Three different species of Elapids can be found in
carbohydrates, lipids, serotonin, and even hista-
North America from North Carolina to Arizona and
mine.18 Local toxic effects include swelling, pain,
as far south as several hundred miles south of the
ecchymosis, and blebs; systemic effects result in
US-Mexican border. The Eastern coral snake
coagulopathy, myocardial injury, muscular paralysis,
(Micrurus fulvius) is the most dangerous of the coral
and central nervous system injury. 4,6 See Table 1 for
snakes. The highest concentrations are found in
a more detailed description of venom effects.
Florida, but it has also been found from North
Crotalid venom has multiple effects on many
Carolina to Louisiana. The Western coral snake
organ systems and one of the most prominent is the
(Micruroides euryxanthus) is found in the Sonoran
venom’s power as an anticoagulant. Recently, it has
Desert of Arizona, northern Mexico, and the south-
been the subject of studies looking into clinical appli-
west corner of New Mexico below 5800 feet. The
cations of a number of proteins isolated from venom
third Elapid in the US is the Yellow-bellied sea snake
for development of new anticoagulant drugs. These
(Pelamis platurus). It has weak venom relative to the
proteins have shown to inhibit platelet adhesion by
other Elapids, but it can still be dangerous to
interfering with the binding of vWF to the GPIb
humans. The range of this sea snake in North
receptor and may provide an entirely new target for
America is limited to the southernmost areas of
antiplatelet agents.19
California and the northern pacific coast of Mexico.
Venom composition will change depending on
Exotic snakes are held by collectors and zoos in scat-
the species of snake, age, diet, geographic location,
tered places throughout the country and may repre-

Emergency Medicine Practice© 4 September 2006 • EBMedice.net


and time of the year.4 Several types of Crotalids, once scorpion antivenom conducted in Tunisia, 82.4% of
mature, have the ability to vary the amount of venom 825 patients had only local effects. The other 17.6%
injected during a strike. Generally, the younger the exhibited systemic effects, primarily autonomic
snake, the more potent their venom.20 instability, e.g. hypertension, sweating, and fever.
Nine (1%) of 825 patients experienced cardiogenic
Coral Snakes shock, six (0.8%) had pulmonary edema, and two
Coral snake venom is a heterogeneous mixture of (0.25%) died.23
peptides and enzymes with primarily neurotoxic
effects on nerve conduction and neuromuscular
transmission.7 The mechanism of action and phar-
macodynamics of coral snake venom is unclear and Section 1. The Pit Vipers (Family Viperidae,
has apparently not been the subject of any published Subfamily Crotalidae): Rattlesnakes, Cotton-
research. It is unlikely that this uncertainty will be mouths (Water Moccasins), And Copperheads
addressed unless, like the Crotalids, a researcher
determines a potential clinical use for Elapid venom
or one of its components. Some cytotoxic effects Identification
may occur but are usually minor especially when
compared to Crotalid venom. Often, there are no Identification of the snake that produced the bite can
symptoms for one to five hours but, once they begin, be extremely helpful. Accurate information about
systemic signs and symptoms may progress rapidly. the identity of a snake can provide information on
Tremors and cranial nerve dysfunction resulting in the potency of the venom (rattlesnake venom being
ptosis, dysarthria, and dysphagia are common in the most potent among native US snakes) and the
substantial envenomations. Respiratory depression patient’s expected clinical course and response to
occurs more rarely and late in the course. antivenom. Often, well-meaning patients or
bystanders will bring in the snake for identification;
Non-native Venomous Snakes it goes without saying that unless you have signifi-
Imported snakes of the family Elapidae share some cant experience handling snakes it is best not to han-
of the characteristics of coral snakes, including a dle specimens that are brought in...alive or dead! If
tendency for elapid venom to cause
Table 1: Snake Venom
mostly neurotoxic effects. Snakes of the
Components And Their Effects
family Viperidae, including Central and
South American and Asian pit vipers
and the true vipers of Africa, the
Middle East, and Europe, are similar
in effect to the Crotalid pit vipers of
North America, with primarily cytotox-
ic venom, resulting in local tissue dam-
age, coagulopathy, and organ dysfunc-
tion.

Scorpions
Scorpion venom is delivered through a
tail stinger from two venom glands and
is a combination of peptides and pro-
teins with proteolytic and neurotoxic
effects. Neurotoxic effects are proposed
to be mediated by effects at sodium and *Refers to quantity of the envenomation.
potassium channels of neurons.21-22 Though not comprehensive, this table is designed to provide an
easy reference in clinical practice.
Most envenomations produce only local
Reprinted from Kiran S, Senthilnathan TA Update in Anesthesia, Issue 16 (2003) Article 6.
effects of pain and swelling. In a ran-
domized, placebo-controlled trial of

EBMedicine.net • September 2006 5 Emergency Medicine Practice©


the animal is brought in dead or in parts, keep in Prehospital / Wilderness Care
mind that snakes retain the ability to strike, bite, and
Multiple methods for treating snakebites in the
potentially cause a significant envenomation for sev-
field have gone in and out of vogue over the years,
eral minutes after death or decapitation. If you have
and have been discussed at length both in the med-
been provided a specimen that is fairly intact and is
ical and non-medical literature. Recently, the
in an appropriate container to allow for safe inspec-
recommended first aid measures for snakebites
tion (and you don’t develop profound cataplexy at
have been revised to exclude many treatments of
the thought of handling a snake), then knowing a
the past. Arterial tourniquets, aggressive wound
few good resources can be helpful in accurate identi-
incision/excision, electric shock, and ice submer-
fication; see Table 5 on page 18.
sion/cryotherapy may all worsen a patient’s
Figure 1 shows some of the characteristic fea-
condition.16-17 Current recommendations for field care
tures that help distinguish the venomous pit vipers
call for very little beyond rapid transportation; see
from the thousands of non-venomous species of
Table 2.
snakes in the US. In general, pit vipers will have a
Field care by bystanders or emergency medical
triangular shaped head, elliptical pupils, and a heat
services should include removing the patient from
sensing pit in front of the eye. These snakes will also
the area where the bite occurred. It is not recom-
have a set of retractable fangs and may or may not
mended to try to catch or kill the offending snake
have a rattle on the tail. The body markings on the
simply to bring it to the hospital. In other words,
Crotalids are widely variable and can be helpful in
common sense and discretion should prevail. A
identifying the specific species of snake. The fourth
snake that has taken up residence under the slide on
species of native venomous snake, the coral snake, is
a school playground is very different from a snake
in a totally different subfamily of snakes and does
that strikes because you stepped on its home in the
not have any of this characteristic set of pit viper fea-
middle of the woods. A good description of the
tures despite possessing very potent venom. The
snake in combination with the patient’s signs and
clinically important feature to know with coral
snakes is the difference in the markings between a
coral snake (venomous) and a king snake (non-ven- Table 2: Prehospital Care of Snake
omous) which has evolved very similar markings to Envenomations
the coral snake in order to fool its prey. The old say-
ing, “red on yellow kills a fellow; yellow on black,
venom lack,” works well to differentiate the corals
from the king snakes in North America ONLY. Both
species are found throughout the geographical distri-
bution of the coral snake.
Generally speaking, if you see more than two or
three snakebites a year or if you are the regional
referral center for snake envenomations (like we
are), you may also want to keep a field guide
specific to your region in your ED reference library.
Peterson Field Guides publishes two titles on
reptiles and amphibians, one for Eastern and
Central North America and one for the Western
US.24-25 These guides are recommended by several
herpetology sources and are inexpensive (approxi-
mately $15 each) references to keep handy. Other
potentially useful resources are poison control cen-
ters, medical toxicology divisions of large hospitals,
state agriculture departments, university veterinary
programs, and local zoos that keep and care for rep-
tiles.

Emergency Medicine Practice© 6 September 2006 • EBMedice.net


Figure 1: Comparison Of Venomous Snakes (Pit Vipers)
And Nonvenomous Snakes In The US

From Gold BS, Dart RC, Barish RA. Bites of venomous snakes. N Engl J Med 2002;347(5):347-58; with permission.

EBMedicine.net • September 2006 7 Emergency Medicine Practice©


symptoms is generally adequate to initiate in-hospi- lancets or scalpels, antiseptic wipes, and various suc-
tal treatment. tion devices. Most of the kits were developed and
Patients should be reassured, placed at rest, kept marketed since before the “incision and suction”
warm, and transported immediately to the closest treatment went out of vogue.
medical facility. The bitten extremity should be The most recognized and researched of these
immobilized and kept at or below the level of the suction devices is the Sawyer Venom Extractor®.
heart, and all constrictive clothing, jewelry and The marketing information available on the internet
watches should be removed.4,15,26 Closely monitor for this product references its ability to remove “up
vital signs to assess for hypotension as a sign of sys- to 30%” of the snake venom based on two small
temic toxicity and hypotension should prompt a studies (one animal in 1985 and one human in 1986)
bolus of intravenous isotonic fluids.5,17 Incision and by Bronstein et al.29-30 A well-designed, 2004 human
suction, once the standard of care for the treatment study by Alberts et al27 using simulated radiolabeled
of snakebites, is no longer recommended and may, in snake venom demonstrated minimal (0.04%) venom
fact, pose more risk than benefit.27 Incision has extraction from a simulated snakebite wound and
never been shown to be beneficial for extracting only a 2% reduction of total body venom load.
venom and can cause unintended and potentially Given the poor performance of this device in the
disabling injury to digital nerves, arteries, and ten- controlled trials and the elimination of most of the
dons even when performed by an experienced former interventions from the field treatment recom-
provider.28 Oral suction of the venom is strongly dis- mendations, Johnson said it best when he stated,
couraged both for its lack of effectiveness in remov- “the best snakebite kit is probably the keys to a car
ing venom and because of the possibility of introduc- that runs.”15
ing oral flora into the wound, potentially complicat- EMS may be faced with a snakebite victim who
ing treatment. has had one or more well-intentioned field interven-
There are several commercial snakebite kits on tions by a bystander and it is important to know
the market containing a multitude of items for the which should be discontinued and which should
treatment of snakebites. Generally there is some remain until hospital evaluation is complete.
combination of venous and/or arterial tourniquets, Incision is one of the most likely field interventions

Table 3: Severity Grading69

Emergency Medicine Practice© 8 September 2006 • EBMedice.net


the EMS provider may encounter. In cases where an omations. Patients with significant envenomations
incision has been made, control bleeding and apply a will have considerable pain both from the local cyto-
moist dressing.17 If an extractor device is correctly in toxic effects of the venom and the diffuse muscle fas-
place it should be left in place until the arrival at the ciculations caused by the neurotoxins and myotoxins
hospital.17 Arterial tourniquets should be removed in Crotalid venom. It is acceptable to give narcotics
due to the potential for limb ischemia but venous and benzodiazepines for comfort while antivenom is
tourniquets or “constriction bands,” defined as wide, being prepared except in the cases of a coral snake,
flat bands that restrict venous and lymphatic flow to Mojave rattlesnake, or Eastern Diamondback rat-
impede absorption of venom, can be left in place. tlesnake envenomation where the neurotoxic effects
These constriction bands, which should be loose of the venom can result in severely impaired mental
enough to allow two fingers to slip easily under- status. There are no good, evidence-based recom-
neath, have shown some benefit in delaying mendations in the literature for choice of drugs or
absorption of venom in experimental models and dosing. NSAIDs should be avoided due to their
have been suggested as therapy in patients with antiplatelet effects potentially worsening venom-
prolonged transport times.17,31 If a device has been induced coagulopathy.
applied and it is not causing vascular compromise, it
should be left in place by EMS during transport. History
EMS personnel should frequently reassess the ten- Once the patient arrives in the ED, key pieces of
sion of any constrictive device to ensure that pro- information pertaining to the snakebite itself must be
gressive limb edema does not result in a venous con- ascertained; see Table 4. Ask the patient about
striction band becoming an inadvertent arterial symptoms that may indicate a significant envenoma-
tourniquet.10 tion; particularly pain, numbness, nausea, tingling
around the mouth, metallic taste, muscle cramps or
ED Management And Stabilization fasciculations, dyspnea, diplopia, or dizziness.4,14-15,18-28
Additionally, ascertain the traditional components of
Initial Stabilization a patient history including a comprehensive review
Upon arrival in the ED, whether by ambulance or of any co-morbid medical conditions (particularly
by personal vehicle, a rapid assessment of the cardiac disease and coagulopathy), a list of current
patient should obviously include an evaluation medications, allergies (especially to papain or
of airway, breathing, and circulation. Barring a papaya based extracts, latex, and horse-based or
situation in which these are compromised and sheep-based products), the time of last oral intake
need to be addressed immediately, the initial stabi- and the patient’s tetanus status. Obtaining the histo-
lization of a snakebite victim includes many of the ry should not interfere with the initiation of treat-
interventions that are recommended for the field ment in the critically ill or clinically deteriorating
providers. patient.
Obtain an initial set of vital signs, place the
patient on continuous cardiac, BP, and pulse oxime-
try monitoring (on an unaffected extremity), remove
constrictive clothing and jewelry, establish intra- Table 4: Key Questions In The History
venous access, and draw blood for labs including Of A Patient With A Snakebite
tubes for a type and screen and coagulation studies.
Supplemental oxygen can be given on a case-by-case • The location of the wound or wounds
basis. Use a Sharpie® or surgical marker to mark
and time the leading edge of erythema. Also, mark • The time of the bite
two to three sites above the bite as locations for serial • The type of snake
measurement of limb circumference.
• Treatment provided by prehospital
Once a clinically significant envenomation has bystander or EMS
been identified, rapidly begin the process of obtain-
• Changes in the patient’s condition since
ing the appropriate antivenom and begin the mixing
the time of the bite
process, and/or arrange for rapid transfer of the
patient to a facility capable of handling snake enven-

EBMedicine.net • September 2006 9 Emergency Medicine Practice©


Physical Exam tration of antivenom), liver function tests, total crea-
Focus the initial physical exam on the evaluation of tine kinase, serum myoglobin, arterial blood gases,
the ABCs and provision of adequate resuscitation. and chest radiography.4,14-15,18,28 This second set of tests
Once the adequacy of all elements of the primary can be ordered judiciously on a case-by-case basis,
survey is established and the steps outlined in the taking into account severity of the envenomation and
initial stabilization are completed, the evaluation of co-morbid disease. Grading severity of envenoma-
the bite site can commence. As mentioned, one- tions will be discussed in detail in the next section.
fourth of snakebites are dry. Examine the site of the
bite for fang marks or scratches and consider the Compartment pressures: The other diagnostic test
possibility of other types of animal bites or injuries if that can be performed in selected cases is the meas-
the diagnosis of a snake envenomation is in doubt. urement of compartment pressures. Most enveno-
Pay particular attention to any local signs of enveno- mations involve only subcutaneous deposition of
mation, i.e. edema, petechiae, ecchymosis, or bullae venom. In the rare intramuscular envenomation,
formation.18 Document circumferential measurements compartment syndrome may develop but it is often
at several sites above and below the bite site.16 Mark impossible to distinguish the symptoms of compart-
a line at the site of each measurement to ensure accu- ment syndrome (classically the 5 “P’s:” pain out
rate reproducibility. Repeat these measurements of proportion, pallor, parasthesia, paralysis, and
every 15 to 30 minutes during the course of treat- pulselessness) from the symptoms of a significant
ment. Also, mark and time the edge of the swelling envenomation. Formerly, fasciotomy was the recom-
to serve as an index of local progression.4 mended treatment for snakebites with suspected
Focus the remainder of the physical exam prima- compartment syndrome, but current review suggests
rily on the cardiovascular, pulmonary, and neurolog- a more conservative treatment plan involving serial
ic systems. Patients may be hypotensive due to third Stryker measurement for compartment pressures.
space losses and hemorrhage. Initial treatment for
hypotension is intravenous isotonic fluids.28 The Treatment
neurologic exam becomes particularly important in
severe envenomations, especially in cases of Mojave General Principles
rattlesnake, coral snake, or non-native/exotic enven- The treatment of patients with snake envenomations
omations where altered mental status and neurologic entails aggressive supportive care and early adminis-
impairment can be a significant and often delayed tration of adequate doses of antivenom when indi-
feature of the envenomation.18 cated.4 Airway, breathing, and circulation are of pri-
mary importance. Establish peripheral intravenous
Diagnostic Studies access in an unaffected extremity. Hypotensive
The number and type of recommended diagnostic patients should receive fluid boluses to replace third
studies in a patient with a snake envenomation space losses and may require blood transfusion if a
varies throughout the snakebite literature. However, venom-induced coagulopathy has resulted in signifi-
the majority of authors agree that a core group of cant hemorrhage. The transfusion of blood products
tests is indicated: A baseline complete blood count will only temporize coagulopathy and ongoing
(CBC) with platelet count, coagulation studies blood losses until the venom is neutralized by
including prothrombin time, partial-thromboplastin antivenom. To be clear...fresh frozen plasma does
time, activated partial-thromboplastin time, fibrino- not fix an envenomation induced coagulopathy.
gen level, fibrin split products (fibrin degradation
products), basic electrolytes, blood urea nitrogen, Analgesics
serum creatinine, and urinalysis. Patients bitten by Consider analgesics part of the standard therapy for
an unknown species with no evidence of toxicity envenomations. Pain control will usually require
require, at a minimum, coagulation studies which parenteral narcotics for the first 24 to 48 hours of
are necessary for the grading of the envenomation. therapy in patients that are receiving antivenom.33
Various sources also recommend obtaining an elec- Skeletal muscle fasciculations can be treated with
trocardiogram, a specimen for type and screen or parenteral benzodiazepines.
type and crossmatch for blood products (as cross-
match may be more difficult following the adminis-

Emergency Medicine Practice© 10 September 2006 • EBMedice.net


Wound Care cally break envenomations into four or five cate-
Even in the absence of a clinically significant enveno- gories, ranging from “dry bite” or nonenvenomation
mation, snakebites are puncture wounds and there- to severe/very severe. Table 3 on page 8 is a synthe-
fore require local wound cleansing and tetanus pro- sis of several scoring systems from different authors
phylaxis. Interestingly, there is good data to show and can be used to grade the envenomations of all
that snakebites have very low rates of infection.34-35 pit viper or suspected pit viper envenomations3.
In spite of a broad spectrum of oral flora cultured Envenomations are graded based on the symptom or
from the mouths of snakes, the venom is postulated sign that places the patient in the highest (most
to have antibacterial properties.36 Based on these severe) category.4 Treat moderate and severe/very
studies, prophylactic antibiotics are not currently rec- severe envenomations (Grades II-IV) with antiven-
ommended for snakebites.33 om. We have combined the Grade III and IV cate-
gories for simplicity since the treatment remains the
Compartment Syndrome same and the signs and symptoms differ only in
In cases with documented pressures above 30mmHg, matter of degree.
give an additional four to six vials of antivenom CroFab™ was FDA approved in 2000 for the
along with 1 to 2gm/kg of mannitol over 30 minutes; treatment of all “mild to moderate North American
limb elevation and re-evaluation of pressures after Crotalidae envenomations;” however, all of the
one hour should occur prior to consideration of fas- research upon which this approval was based was
ciotomy.4,28,32 It is recommended that the appropriate done with rattlesnake envenomations.37,40
service that handles fasciotomy in your facility (gen- Copperheads, while a member of the Crotalidae sub-
erally, surgery or orthopedics) be consulted early in family of vipers, are a separate species from the rat-
the course of treatment despite the rarity of enveno- tlesnakes and copperhead venom is not used in the
mations that ultimately require this procedure. preparation of CroFab™. As recently as 2004,
authors have recommended against the administra-
Crotalidae Polyvalent Immune Fab (CroFab™) tion of CroFab™ for copperhead bites because the
CroFab™ was approved by the FDA in October of venom effects were not “serious enough” and the
2000. CroFab™ is a preparation of ovine Fab (mono- risks of antivenom outweighed the benefits.40 One
valent) immunoglobulin fragments obtained from the retrospective review of copperhead bites by a group
blood of healthy sheep flocks. These sheep are at Carolinas Medical Center showed that CroFab™
immunized with one of the following North administration resulted in an improvement in local
American snake venoms: Crotalus atrox (Western symptoms.41 Unfortunately, this was a small study
Diamondback rattlesnake), Crotalus adamanteus involving only 32 patients. The selection criteria for
(Eastern Diamondback rattlesnake), Crotalus scutula- treating with antivenom were not standardized and
tus (Mojave rattlesnake), and Agkistrodon piscivorus the “treatment group” represented only 8% of the
(Cottonmouth or Water Moccasin).37 Note that cop- copperhead envenomations that presented to
perhead venom is not included in the immunizations. Carolinas Medical Center during the review period.
According to the CroFab™ package insert, A larger, multicenter, randomized controlled study of
patients with allergies to papain, chymopapain, other CroFab™ in the treatment of copperhead bites is cer-
papaya extracts, or the pineapple enzyme bromelain tainly warranted and needs to include comparisons
may be at risk for an allergic reaction to CroFab™. In not only of local symptoms, but also of short- and
addition, it has been noted in the literature that some long-term disability and adverse events associated
dust mite allergens and some latex allergens share with treatment versus non-treatment.
antigenic structures with papain and patients with After the severity of the envenomation has been
these allergies may be allergic to papain.38-39 determined, patients with an indication to receive
CroFab™ should have the drug administered as soon
Indications: Initiating treatment of an envenomation as possible. Early (< 6 hrs) administration of
with CroFab™ is largely based on physical findings, CroFab™ has been clinically shown to reduce clinical
their severity, and development over time. decline and systemic coagulopathy. Reassess and
Classically, envenomation severity has been deter- admit these patients to the ICU for monitoring and
mined based on symptoms and quantified using one the completion of the CroFab™ treatment.
of a few accepted grading scales. These scales typi- When the possibility of an envenomation by a

EBMedicine.net • September 2006 11 Emergency Medicine Practice©


coral snake or Mojave rattlesnake exists, the scale reconstitution of CroFab™ can begin prior to patient
presented in Table 3 should not be used. The signs arrival. Nonetheless, the eagerness to begin the
and symptoms of such venom may be significantly process of mixing needs to be tempered by the possi-
delayed and have their primary effects on the nerv- bility of not needing to use a treatment which has an
ous system. In these cases, extend the observation average cost of nearly $1000 per vial.
period to at least 12 hours from the time of the injury The recommended initial dose of CroFab™ is
to account for this delay.4-5,18 Confusing matters four to six vials. Dosage requirements are dependent
somewhat, some authors recommend admission of upon the individual patient response. The use of the
all suspected coral snake envenomations for close recommended adult dosages in the pediatric popula-
monitoring.4 Presumably the authors are differentiat- tion appears to be safe.43-44 The initial dose should
ing “observation” in the ED from “admission” mean- not be reduced in a pediatric patient because, despite
ing 23 hours or more. Unfortunately, there is no their smaller size, the volume of venom to be neu-
good clinical data upon which these recommenda- tralized is not reduced. Anecdotal experience with
tions are based. the older ACP venom indicates that antivenom
requirements may, in fact, be higher in pediatrics; but
Preparation and Administration: If possible, obtain no clear clinical correlation between age, size,
written, informed consent prior to the administration weight, or snake species and antivenom requirement
of CroFab™, advising the patient primarily of the has ever been demonstrated with either antivenom
risks of anaphylaxis, delayed serum sickness, and/or preparation. However, the fluid status of the pedi-
death. In the initial clinical studies of CroFab™, the atric patient should be taken into account when
rates of allergic reaction were low and skin testing
deciding on the initial dilution of the dosage to be
was not done. Skin testing done in the setting of the
given. Most children can handle the 250cc volumes
older polyvalent antivenom administration (which
without difficulty as this correlates to a 20cc/kg
had a much higher anaphylaxis rate) has been shown
to have both high false-positive rates and false-nega- bolus for a 12.5kg child, but fluid status may become
tive rates, 33% and 10 to 36% respectively. As clini- an issue in children less that 10kg.44 Unfortunately,
cal experience and comfort with CroFabTM have there are no controlled studies to provide guidance
increased, the need for skin testing prior to adminis- on the delivery of higher concentrations.
tration has never been shown to be useful. Deliver the initial dose at a rate of 25 to 50mL/hr
Additionally, skin testing delays the onset of defini- for the first ten minutes. If no acute allergic reaction
tive treatment and the risk of true anaphylaxis with is noted after this initial time; deliver the remainder
CroFab™ is substantially lower than with the older of the first four to six vial dose at the full rate of
Antivenom Crotalid Polyvalent (ACP).42 Despite this 250mL/hr. If a reaction occurs, administer both H1-
data, the treating physician should always be pre- and H2-receptor blockers. If the symptoms resolve
pared for an anaphylactic reaction to antivenom with
and the reaction was mild, the infusion can be con-
basic supplies such as epinephrine, diphenhy-
tinued with close monitoring. In a retrospective
dramine, airway equipment, oxygen, and pressors.
review of CroFab™ safety by Dart and McNally, the
CroFab™ is packaged as a shelf-stable
two patients with severe reactions (cough and wide-
lyophilized powder in a vial. Reconstitute each vial
spread urticaria +/- wheezing) had infusions dis-
of CroFabTM with 10mL of Sterile Water for Injection
continued, diphenhydramine and an H2-receptor
USP (diluent is not included). After being thorough-
blocker given, and then infusions restarted. In one
ly mixed, further dilute each of the reconstituted
patient the reaction recurred and no further antiven-
vials to be used in a given dose in a single 250mL
om was given. In the other patient, the infusion was
bag of 0.9% Sodium Chloride USP. Use this reconsti-
restarted and completed without incident with the
tuted and diluted product within four hours of mix-
precaution of an epinephrine infusion which is only
ing. The reconstitution and dilution process is time
described as being administered “in standard
consuming; even in the most practiced hands, prepa-
doses”45-46 or as a “low dose infusion” which was
ration of an initial dose of CroFab™ takes more than
stopped 30 minutes after the completion of the infu-
30 minutes. Given this, our recommendation is that
sion of CroFab™.47
the mixing of CroFab™ should begin as soon as the
After the initial dose of four to six vials, “initial
patient demonstrates evidence of a significant enven-
control” is achieved when an adequate clinical
omation. Occasionally, in select scenerios, when
response has occurred, i.e. no further progression of
patients are being transported for treament, the

Emergency Medicine Practice© 12 September 2006 • EBMedice.net


any local symptoms, systemic symptoms, or coagu- incidence and severity of reactions to CroFab™
lopathy. Observe the patient while administering appears to be significantly reduced. In the initial
two additional vials every six hours for three addi- clinical studies discussed above, seven of the first 42
tional doses. If, after completion of the initial dose, patients treated had an early, though relatively
the patient’s symptoms continue to progress (wors- minor, reaction (five urticaria, one cough, one
ening of the local injury or systemic effects such as urticaria/dyspnea and wheezing). However, it is
muscle fasciculation, parasthesias, abnormal mental important to note that there have been case reports
status, tachypnea, tachycardia, or hypotension), an of serious reactions and serum sickness related to
additional four to six vials is recommended. This CroFab™.46-47
additional dose is also warranted if laboratory stud-
ies show prolonged coagulation times, decreasing Disposition
fibrinogen levels, or worsening platelet count. If a
second four to six vial dose is given, the patient Patients who have no findings of envenomation (dry
needs to be reassessed for “initial control” and a bites/Grade 0) can be discharged after a four to six
third round of four to six vials may be needed in the hour observation period.18 Patients who have limited
most severe cases. Multiple studies of CroFab™ local signs of envenomations but do not meet the cri-
indicate that the dose required for initial control can teria for CroFab™ need to be monitored for progres-
range from 4 to 18 vials. If three rounds of four to sion of symptoms in the ED or observation unit for
six vials fail to achieve control of an envenomation, a minimum of 8 to 12 hours from the time of the
consider an alternative diagnosis, such as another bite4-5,18,28 Although most patients with Crotalidae
type of envenomation, a toxic overdose, or an exotic envenomations begin to manifest local symptoms
species. within 10 minutes of a bite and virtually all have
The elimination half-life for CroFab™ is estimat- some findings within 30 to 60 minutes, there are some
ed to range from 12 to 23 hours.37 Further study has patients who will not manifest symptoms for several
shown that Fab molecules have a shorter half-life hours, although the mechanism for this is unclear.4 In
than the IgG used in traditional polyvalent antiven- those cases where the patient does not initially meet
om.45 Early studies of CroFab™ reported cases of treatment criteria, observation and intermittent
recurrence of signs and symptoms of the original reassessment is critical since local findings or symp-
envenomation after intital control. This may be due toms that would upgrade the envenomation into a
to a Fab half-life that is shorter than the elimination treatment category may be absent initially.
half-life of snake venom, thus the administration of Patients who receive CroFab™ are admitted to
CroFab™ includes three additional doses to prevent the hospital as discussed in the previous section. The
symptom recurrence. Once initial control is half-life of CroFab™ is less than that of polyvalent
obtained, admit the patient to the ICU and give two antivenom. This fact, coupled with the persistent
vials of CroFab™ at 6, 12, and 18 hours after the activity of the depot-style delivery of the venom, can
completion of the control doses. lead to recurrent coagulopathy despite appropriate
antivenom treatment.68 Such recurrence is character-
Monitoring: After stabilization and initial control of ized by decreased platelets, elevated prothrombin
the patient’s symptoms have been achieved, and the time, and decreased fibrinogen. During the initial
administration of the scheduled doses has begun, trials, such a recurrent coagulopathy was observed
continue close surveillance of the patient’s condition. only in those patients experiencing coagulation
Observation includes monitoring of limb circumfer- abnormalities at initial presentation. After initial
ence, both above and below the bite, and using a pen treatment, this coagulopathy may be present for
to outline the edematous area every 30 to 60 minutes. weeks. Therefore, post treatment, monitor those
Obtain laboratory determinations of the patient’s patients presenting with coagulopathy for signs and
coagulation status every four hours.28 If symptoms symptoms of recurrent coagulopathy after discharge
or laboratory data warrants, the treating physician every two to three days until the coagulopathy
can give an additional two vials. Any unscheduled resolves48-49 Advise patients to contact their physician
re-dosing after initial control does not reset the 6, 12, immediately if they experience new symptoms or
18 hour schedule to the beginning. unusual bruising or bleeding after hospital discharge
When compared to polyvalent antivenom, the as additional antivenom treatment may be needed.

EBMedicine.net • September 2006 13 Emergency Medicine Practice©


Emergency Medicine Practice© 14 September 2006 • EBMedice.net
EBMedicine.net • September 2006 15 Emergency Medicine Practice©
Emergency Medicine Practice© 16 September 2006 • EBMedice.net
Boyer et al recommend retreatment with two vials toms of compartment syndrome and it is clinically
of CroFab™ in the event of fibrinogen level impossible to establish the difference without direct
< 50mcg/mL, platelet count < 25,000, INR > 3.0, measurement of compartment pressures.53 Even in
aPTT > 50 seconds, multicomponent coagulopathy, the cases where compartment pressures exceed 30 to
worsening trend in patients with prior severe coagu- 40mm Hg, additional antivenom administration
lopathy, high risk behavior for trauma, or comorbidi- (four to six vials) has been shown to reduce compart-
ties that increase the risk of hemorrhage.49 ment pressures, avoiding an unnecessary and disfig-
Instruct patients to report any signs or symp- uring fasciotomy. Fasciotomy should only be consid-
toms of delayed allergic reactions or serum sickness ered in the extraordinarily rare cases where addition-
after hospital discharge. Serum sickness is a delayed al antivenom doses have failed to reduce measured
hypersensitivity reaction to antivenom characterized pressures.
by fever, rash, arthralgias, and lymphadenopathy In the authors’ opinion, given the ”assess, treat,
which typically begin 7 to 21 days following antiven- reassess, treat” management strategy, and the sched-
om administration. It is much more rare with uled dosing nature of current management, a medical
CroFab™ than with the older polyvalent antivenom ICU service may be more appropriate than a surgical
(3% versus more than 80%) and responds well to a service. Clearly, in the rare case of true elevated com-
tapering course of prednisone starting at 60mg daily partment pressures, a consultation to orthopedics or
and tapering over seven to ten days.4,33,45 Serum sick- surgery is warranted. That said, snake envenoma-
ness is the only indication from steroids in the treat- tions present such a complex interaction of multisys-
ment of snake envenomation. tem toxicologic effects that the determination of the
best admission service should be handled on a case-
Controversies by-case and hospital-by-hospital basis.

Alternative Treatment
Several studies have addressed specific techniques
used in the care of snake envenomations and have Section 2. Coral Snakes (Elapidae), Exotic
shown that many of the devices and techniques tra- Snakes, And Scorpions
ditionally used in care of snake envenomations were
either of no benefit or were, in fact, harmful. Prehospital Care
Incisions, suction devices, packing in ice (cryothera-
py), application of heat, and even application of elec- As with Crotalid envenomations, there are no specif-
trical shocks have all historically been used to treat ic prehospital interventions for scorpion or non-
snake envenomations. While the application of a Crotalid snake envenomations, other than removal
stun-gun or “gasoline engine spark plugs” to a from danger, and rapid, safe transport. Immobiliza-
patient may be intriguing, there is no clinical benefit, tion of the affected limb may be helpful.
and a handful of case studies show the danger of this An interesting experiment testing a novel immo-
therapy, even in controlled settings.50 As discussed bilization technique in a porcine model of a coral
in detail above, little role remains for any field treat- snake envenomation has been published.8 The pres-
ment beyond immobilization, reassurance, and rapid sure-immobilization technique is not a tourniquet,
transport. but uses an elastic bandage applied from the enveno-
mation site and extended proximally. The goal is to
Fasciotomy impede lymphatic flow, not venous or arterial flow.
Once considered part of the primary treatment of Apply the bandage about as tight as a wrap for an
snake envenomations, fasciotomy is the other contro- acute sprain, yet loose enough to allow a finger to be
versial issue that has yet to be addressed in this dis- inserted between skin and bandage without difficul-
cussion. Good clinical data from animal studies ty. Also, splint the limb to limit motion. Elapid
have shown that in snake envenomations, rabbits snakebites in other countries, primarily Australia, are
that receive fasciotomy (with or without antivenom) treated with a similar pressure immobilization tech-
have poorer outcomes than those treated with nique.54-55 Because of the low quality evidence, this
antivenom alone.51-52 The signs and symptoms of a technique must be considered experimental, even
significant envenomation closely mimic the symp- though it has been included in a clinical guideline
for the snakebite section of an emergency care guide-

EBMedicine.net • September 2006 17 Emergency Medicine Practice©


line, the 2005 International Consensus Conference on occur over a few minutes. Tremors, ptosis, dyspho-
Cardiopulmonary Resuscitation and Emergency nia, dysphagia, and decreased deep tendon reflexes
Cardiovascular Care Science with Treatment are the most common neurologic signs and may indi-
Recommendations.11 cate impending respiratory paralysis.7

Emergency Department Evaluation Non-native Venomous Snakes


Depending on the species, envenomations may pro-
Coral Snakes duce mostly neurotoxic effects, especially from the
Coral snakebites produce little, if any, pain and local other Elapids, e.g. cobras, kraits, and mambas.
reaction. Assessment of signs of neuropathy must be Cytotoxic and coagulopathic effects may predomi-
aggressively sought, as progression to bulbar paraly- nate from bites of other snakes, especially the
sis and respiratory failure may be rapid. The onset Viperidae (both pit and true vipers). Once identifica-
of neurological symptoms may be delayed one to tion is made, a poison center consultation and/or
four hours, but once neurological signs appear, pro- accessing one of the available web resources will
gression to paralysis and respiratory failure may help guide clinical decision making; see Table 5.

Table 5: Useful Websites For Snake


Identification And Bite Management
Black and white photographs of snakes do very little to help in the identification of snakes and thus, are not reproduced in this article.
Fortunately, the internet has proven to be extremely useful for snake identification with many sites that provide a variety of full color
pictures and tools to help identify venomous snakes and differentiate them from their non-venomous counterparts. Using a search
engine and the topic "snake" or "snake identification" will result in websites that discuss the snakes that are indigenous to your region
and will aid in identification. This table lists some of the useful websites for snake identification and bite management.

• http://mdg.ext.msstate.edu/Tom_Snake/ unknown snake, but if you have a good idea


index.html: One of the most user friendly sites of what you are looking for, this is an excellent
as it has been designed for identification of site for quick confirmation.
snakes by the non-herpetologist. Snakes are
identified by name or features. It is based at • http://trailquest.net/SNpoi.html: Avid hikers will
Mississippi State University and will identify the recognize this site as an excellent resource for all
snakes of the Southeast and the majority of the things related to hiking. It also happens to con-
snakes nationwide. tain a complete list of "poisonous" and "nonpoiso-
nous" snakes that may be encountered while hik-
• www.snakesandfrogs.com: This site provides ing throughout the continental United States.
excellent descriptions (with photos) of the Like the previous site, it requires that you choose
various characteristics that identify snakes. a species to see a photo, but it does have better
Follow the links to "How to Identify Snakes." descriptions of habitat and distributions of species
It is a South Carolina based site that will work by state/region.
for most snakes indigenous to the US East and
Southeast. It is an excellent site if you have • www.aza.org: The American Zoo and Aquarium
the specimen in front of you but tougher to navi- Association (AZA) can be particularly helpful
gate if you are working solely from a patient’s when exotics are involved in an envenomation.
description. The AZA maintains the Antivenom Index, a list of
the types and locations of all species-specific
• http://www.pitt.edu/~mcs2/herp/SoNA.html: antivenoms for any of the venomous exotics
This site’s common name is "Snakes of North housed in US zoos and larger private collections.
America" and it can be found easily by searching The Antivenom Index is a members-only list, but
"snake identification" on Google. It is a compre- national and regional Poison Control Centers
hensive list of every snake native to North maintain active memberships and can access the
America with a picture. Each snake only has information needed to provide patient care
one photo and requires that you choose the (American Association of Poison Control Centers,
species in order to see the photo. This can www.aapcc.org, 800-222-1222).
prove to be a challenging way to identify an

Emergency Medicine Practice© 18 September 2006 • EBMedice.net


Scorpions Non-native Venomous Snakes
Children are at the greatest risk of severe effects Again, guidance on testing will depend on the iden-
because of low body weight to venom ratios. Elderly tification of the snake. Bites from other members of
people may also have lower reserves against the the family Elapidae will likely require no testing.
physiologic insult of the toxins. Evaluation of local, Patients with bites from other Viperidae snakes will
systemic, and neurological effects is essential. probably require testing similar to that recommend-
Deaths occur from cardiopulmonary collapse or res- ed previously for Crotalid envenomations, e.g. com-
piratory paralysis. Identification of impaired tissue plete blood count, comprehensive metabolic panel,
perfusion, tachycardia, tachypnea, hypoxemia, prothrombin time, partial thromboplastin time, fib-
hypotension, agitation, altered mental status, and/or rinogen level, blood type and antibody screening.
cranial and somatic neuromuscular dysfunction are a Compartment pressure measurement is indicated if
prompt for aggressive treatment. A retrospective compartment syndrome is suspected.
study of 428 patients admitted to the intensive care
unit in Tunisia for scorpion envenomations found Scorpions
that respiratory rate > 30 breaths per minute, agita- A complete blood count and plasma protein concen-
tion, and sweating were predictors of pulmonary tration may help predict the presence of pulmonary
edema.56 edema, as will a chest x-ray. Cardiac enzymes may
demonstrate cardiac damage. Liver enzymes may
Diagnostic Studies show liver damage which was correlated with poor
prognosis in a study of 951 patients admitted to a
Coral Snakes Tunisian ICU.57 Echocardiography may be necessary
There is no adequate evidence to support the use of to evaluate cardiogenic shock or suspected cardiac
any specific laboratory or imaging tests in cases of dysfunction.
coral snake envenomations. Since there are generally
few cytotoxic effects, there is probably no need for Treatment
testing. However, if the patient requires endotra-
cheal intubation and mechanical ventilation for res- Prompt administration of specific antivenom, when
piratory failure, a chest x-ray to assess tube position available and indicated, and supportive care of air-
and arterial or venous blood gases to assess ventila- way, breathing, circulation, and neurologic function
tion status are indicated. are the foundation of ED care for these envonoma-
tions and stings. Tetanus prophylaxis should be

Key Points

1. Venomous snakes (domestic and imported) and 5. Zoos maintain a stock of antivenoms for many
scorpions can produce devastating injuries and exotic, venomous snakes and may be a source for
must be recognized promptly and treated appro- treating envenomations from those snakes.
priately to prevent morbidity and death.
6. Your local poison control center has access to mul-
2. Identification of the snake that inflicted the bite is tiple sources for identifying venomous snakes
important but not essential for appropriate man- and obtaining specific antivenom. Call them.
agement
7. Children and elderly are at the highest risk of
3. Crotalid envenomations are graded based on the severe morbidity and death from scorpion stings
most severe sign or symptom, and CroFab™ and require close observation.
should be administered for all moderate and
8. Equipment and medications for management of
severe envenomations.
anaphylaxis should be in place during the admin-
4. Once a coral snake bite is confirmed, antivenom istration of any antivenom.
should be given immediately, even if no symp-
toms are present.

EBMedicine.net • September 2006 19 Emergency Medicine Practice©


offered if there is no clear history of tetanus immu- Polyvalent (ACP), 56% experienced a rash 3 to 21
nization within five years and encouraged if it has days after antivenom with several experiencing sub-
been more than ten years. Critical care services jective fever, itching, and arthralgias as well. These
including mechanical ventilation, pressor and findings appeared dose related with nearly all
inotrope administration, intravenous hydration patients receiving 30 or more vials of antivenom
and nutrition, and prolonged sedation are sometimes experiencing a rash. Serum sickness, discussed previ-
required in severe envenomations. No credible evi- ously, is much more common with the horse serum
dence supports the use of prophylactic antibiotics or derived antivenoms used for coral snake, exotic
steroids in scorpion, coral snake, or non-native ven- snake, and scorpion envenomations than with
omous snake envenomations. Consideration of spe- CroFab™.4 In a prospective observation study of
cial needs for individual cases is discussed later in 116 patients receiving scorpion (Centruroides)
this article. antivenom for severe envenomations, four patients
Adverse effects of antivenom administration, had immediate reactions: Three cases of rash and one
often anaphylaxis, occur in the majority of patients case of anaphylaxis. Follow-up of the 99 patients was
who receive antivenom derived from animal serum. conducted at one year; 61% experienced delayed
Universal preparation to treat these reactions is hypersensitivity reaction and serum sickness, which
required for all patients receiving antivenom. Two responded to steroids and antihistamines.60
prospective case series, one from Australia and one A prednisone taper, beginning at 60mg per day
from South Africa, found over a 70% occurrence of over seven to ten days, along with oral antihista-
immediate hypersensitivity reactions, with about mines, is the most common treatments for serum
half of these anaphylaxis, after administration of sickness.61 One randomized, controlled trial and a
(non-Fab derived) snake antivenom.58-59 In a study of systematic review of that trial concluded that the
181 patients who received Antivenom Crotalid administration of 0.25mg of 1:1000 epinephrine sub-

Emergency Medicine Practice© 20 September 2006 • EBMedice.net


cutaneously in the forearm immediately before before and after depletion of stocks of antivenom,
antivenom infusion is started markedly reduced the suggest that use of antivenom in children may pre-
incidence of immediate hypersensitivity reactions; vent some hospital and intensive-care unit admis-
absolute risk reduction 30%, number-needed-to-treat sions.64-65
was 3.3, with no significant adverse effects attributa-
ble to epinephrine.62-63 Controversies/Cutting Edge

Coral Snakes A provocative pilot study was conducted in children


Place coral snake specific antivenom at the bedside admitted to the hospital for a scorpion envenoma-
of all patients with a suspected coral snake bite. tion, using a before-after, quasi-experimental analysis
Observe and treat at the first sign of envenomation, of an intervention using prazosin, an alpha-receptor
however minor. The antivenom is derived from blocker. Unfortunately, this study was done in
horse serum and may result in an immediate or Tunisia, involving severe envenomations by North
delayed hypersensitivity reaction. The incidence data African scorpions and a control therapy using
for adverse reactions is not available. insulin and dextrose which is not standard in the US.
Prazosin dosing was 30 micrograms/kg/dose orally
Non-native Venomous Snakes at the time of presentation and repeated three hours
Early and sufficient antivenom administration is the after the first dose and every six hours thereafter as
key to treatment of non-native venomous snakebites. needed to alleviate signs of autonomic instability.
Again, positive identification of the snake is essential Before beginning the prazosin intervention “stan-
so that specific antivenom can be obtained. Local dard therapy” was to use an insulin and dextrose
zoos are required to store antivenom, when avail- mixture, intravenous fluids, and treatment of associ-
able, for every venomous species in their collection. ated complications. Standard therapy was given in
Call your local zoo for availability of antivenom or addition to prazosin. With 20 patients in the “before
call the American Zoo and Aquarium Association cohort” and 16 patients in the “after cohort,” the
(301-562-0777) for access to their antivenom index. authors reported that the prazosin group experi-
Collectors of venomous snakes are not bound by enced fewer episodes of hypoglycemia and hyper-
these same regulations, so a search for antivenom kalemia; although, it is unclear if this is an effect of
must be conducted, after identification of the snake, the scorpion venom or a side effect of the
through local poison control centers or the American insulin/dextrose therapy. Only one of 16 patients in
Association of Poison Control Centers (800-222-1222). the prazosin group died, compared to seven of 20 in
the control group. Hospital length-of-stay was
Scorpions reduced in the prazosin group from a mean of 71.5
Supportive care is the cornerstone of treatment. hours to 46.3 hours.66 With the unavailability of scor-
Scorpion antivenom for Centruroides sculpturatus (aka pion antivenom in Arizona, there is a pressing need
exilicauda), the only scorpion species in the US dan- for a well designed, randomized, controlled trial of
gerous to humans, was previously available only in prazosin in the treatment of scorpion envenomations
Arizona, but production stopped in 2001. Stocks with systemic signs.
became outdated in 2004, and are not FDA approved A randomized clinical trial of scorpion antiven-
for the treatment of scorpion envenomations. In om is currently being conducted. A two-year clinical
addition, there are contradictory findings from trial of 50 patients, supported by a FDA grant to the
studies of the effectiveness of antivenom in treating University of Arizona, to study the effect of a new
scorpion stings. Complicating this, is the fact that scorpion FAB fragment antivenom has been complet-
most studies have been done on non-North ed but was only reported in a Tuscon, Arizona news-
American scorpion envenomations. For example, paper.67 The drug, Anascorp™, is made in Mexico
one randomized, placebo-controlled trial in Tunisia and not yet approved for use by the FDA. The clini-
and a systematic review including the randomized cal trial overseen by researchers at the University of
trial and three cohort studies concluded that there Arizona and the Arizona Poison Center in Tucson
was no benefit to administering antivenom.9-10 More reported that the drug used in > 100 Arizona chil-
relevant to US practice, but weaker evidence, a case dren demonstrated benefit.
series and a natural before-after analysis of cohorts

EBMedicine.net • September 2006 21 Emergency Medicine Practice©


Disposition Case #1: This case represents a very severe enveno-
mation (Grade III/IV). This patient needed 12 vials of
Coral Snake
CroFab™ to achieve initial control and resolve the coagu-
Any time a patient with a coral snake envenomation
lopathy. He was admitted to the medical ICU and moni-
exhibits severe systemic or neurological signs, admit
tored for several days. His pain was controlled with nar-
the patient to the intensive care unit. If the patient
cotics and benzodiazepines; he ultimately did well and was
does not exhibit any systemic symptoms or signs at
discharged home. He did not get a new pair of boots.
the end of a 12 hour ED observation period, the
Case #2: This patient required eight hours of observa-
patient may be discharged home.4
tion. The bite was a “Grade 0” at presentation and he
If the patient has systemic signs after four hours
remained asymptomatic for the eight hour stay. The
and is requiring any supportive care, admit the
patient was discharged home after wound cleansing and
patient for at least a 24 hour observation period. At
administration of tetanus prophylaxis.
the end of the observation period, patients no longer
Case 3: In this case, the patient was experiencing
requiring supportive care may be discharged.
neurotoxic effects of a coral snake envenomation and was
Patients with continuing systemic signs and support-
at a high risk for respiratory failure. Three vials of North
ive needs should continue to receive antivenom and
American coral snake antivenom were given, with addi-
be managed in an ICU setting.
tional doses made available at the bedside. Aggressive
supportive care, including endotracheal intubation and
Non-native Venomous Snakes
mechanical ventilation were required. The patient made an
Because of the wide variety of snakes and venom
uneventful recovery. He decided to hire someone the next
effects, no firm disposition criteria can be deter-
time the brush needed clearing.
mined. Consult a poison control center to assist in
medical management decisions.68
References
Scorpions Evidence-based medicine requires a critical appraisal
Any time a patient with a scorpion envenomation of the literature based upon study methodology and
exhibits severe systemic signs, admit the patient to number of subjects. Not all references are equally
the intensive care unit. If the patient does not exhibit robust. The findings of a large, prospective, random-
any systemic symptoms or signs at the end of a four ized, and blinded trial should carry more weight
hour ED observation period, the patient may be dis- than a case report.
charged home with no specific discharge instruc- To help the reader judge the strength of each ref-
tions. If the patient has systemic signs or requires erence, pertinent information about the study, such
supportive care, admit for at least a 12 to 24 hour as the type of study and the number of patients in
observation period. At the end of the observation the study, will be included in bold type following the
period, patients no longer requiring supportive care reference, where available.
may be discharged.
1. Watson WA, Litovitz TL, Klein-Schwartz W, et al. 2003
Conclusion Annual Report of the American Association of Poison
Control Centers Toxic Exposure Surveillance System.
There are approximately 8000 venomous snakebites Am J Emerg Med 2004;22(5):335-404. (Prospective;
in the US each year. The vast majoity of these bites 2,395,582 patients)
are from rattlesnakes, copperheads and water 2. Watson WA, Litovitz TL, Rodgers GC, et al. 2004
mocosins. Management is based on early recogni- Annual Report of the American Association of Poison
Control Centers Toxic Exposure Surveillance System.
tion, envenomation assessment, and administration
Am J Emerg Med 2005;23(5):589-666. (Prospective;
of the appropriate antivenom. Current concepts in
2,438,644 patients)
care have relenquished incision and suction tech- 3. Langley RL, Morrow WE. Deaths resulting from ani-
niques to history books and have promoted antiven- mal attacks in the United States. Wilderness Environ
om that is immunotherapy based. Med 1997;8:8-16. (Expert opinion)
The three cases presented at the beginning of this 4. Gold BS, Barish RA, Dart RC. North American Snake
article illustrate scenarios that may confront any Envenomation: diagnosis, treatment and management.
emergency medicine physician. The case outcomes Emerg Med Clin North Am 2004;22(2):423-43. (Review)
5. Gold BS, Dart RC, Barish RA. Current Concepts: Bites
show the benefit of proper clinical management.

Emergency Medicine Practice© 22 September 2006 • EBMedice.net


of Venomous Snakes. N Engl J Med 2002;347: 347-356. 22. Rodriguez de la Vega RC, Possani LD. Current views
(Review) on scorpion toxins specific for K+ channels. Toxicon
6. Dart RC, Waeckerle JF. Introduction: “Advances in the 2004;43(8):865-875. (Basic science)
Management of Snakebite” Symposium. Ann Emerg 23. Zuo XP, Ji YH. Molecular mechanism of scorpion neu-
Med 2001;37:166-167. (Expert opinion) rotoxins acting on sodium channels: insight into their
7. Kitchens CS, Van Mierop LH. Envenomation by the diverse selectivity. Mol Neurobiol 2004;30(3):265-278.
Eastern Coral Snake (Micrurus fulvius fulvius). A (Basic science)
study of 39 victims. JAMA. 1987;258(12):1615-1618 24. Conant R, Collins JT. Reptiles and Amphibians:
8. German BT, Hack JB, Brewer K, et al. Pressure-immo- Eastern and Central North America 3rd ed. New
bilization bandages may delay toxicity in a porcine York: Houghton Mifflin; 1998. Peterson Field Guide
model of eastern coral snake (Micrurus fulvius fulvius) Series. (Reference book)
envenomation. Ann Emerg Med 2005;45(6):603-608. 25. Stebbins RC. A Field Guide to Western Reptiles and
(Animal study, randomized, controlled) Amphibians 3rd ed. New York: Houghton Mifflin;
9. Abroug F, El Atrous S, Nouira S, et al. Serotherapy in 2003. Peterson Field Guide Series. (Reference book)
scorpion envenomation: a randomized controlled trial. 26. Venomous Snakes of North America. Accessed 12
The Lancet 1999;354:906-909. (Prospective, random- September 2006 at http://www.geocities.com/
ized, controlled; 825 patients) RainForest/2221/snakeid.html.
10. Foex B. BestBETs: Scorpion envenomation: Does 27. Alberts M, Shalit M, LoGalbo F. Suction for Venomous
administration of antivenom alter outcome? Accessed Snakebite: A Study of “Mock Venom” Extraction in a
25 May 2006 at http://www.bestbets.org/cgi- Human Model. Ann Emerg Med 2004; 43(2) 181-186.
bin/bets.pl?record=00567 . (Systematic review) (Prospective;8 human volunteers)
11. ECC Committee, Subcommittees and Task Forces of 28. Dart RC, Daly FS. Reptile Bites. In: Tintinalli J, editor.
the American Heart Association. First aid: 2005 Emergency Medicine: A Comprehensive Study Guide,
International Consensus Conference on 6th ed. Chicago: McGraw Hill; 2004. p1200-1205.
Cardiopulmonary Resuscitation and Emergency (Textbook Chapter)
Cardiovascular Care Science with Treatment 29. Bronstein AC, Russell FE, Sullivan JB. Negative pres-
Recommendations. Circulation 2005;112(22 Suppl):15- sure suction in the field treatment of rattlesnake bite.
25. (Consensus conference report) Vet Hum Toxicol 1985;28:297 (Abstract)
12. Parrish HM. Incidence of treated snakebites in the 30. Bronstein AC, Russell FE, Sullivan JB. Negative pres-
United States. Public Health Rep 1966;81:269-276. sure suction in the field treatment of rattlesnake bite
(Survey) victims. Vet Hum Toxicol 1986; (Abstract)
13. Russell FE, Banner W. Snake venom poisoning. Conn’s 31. Burgess JL, Dart RC, Egen NB, et al. Effects of con-
Current Therapy. Rakel RE (ed). Philadelphia, WB striction bands on rattlesnake venom absorption: a
Saunders Co, 1988, p 1002. pharmacokinetic study. Ann Emerg Med 1992;21:1086-
14. Juckett G, Hancox JG. Venomous Snakebites in the 1093.
United States: Management Review and Update. Am 32. Hall EL. Role of surgical intervention in the manage-
Fam Phys 2002;65(7):1367-1374. (Review) ment of crotaline snake envenomation. Ann Emerg
15. Johnson CA. Management of Snakebite. Am Fam Med 2001;37(2):175-180. (Review)
Phys 1991;44(1):174-180. (Review) 33. Singletary EM, Rochman AS, Bodmer JC, et al.
16. Liebelt EL, Kazzi ZN. Timing and Toxicology Critical Envenomations. Med Clin N Am 2005; 1195-1224.
for Treating Snakebite Victims. UAB Insight 2006; (Systematic review)
Spring: 9-10. (Expert Opinion) 34. Clark R.F, Selden BS, Furbee B, The incidence of
17. McKinney PE. Out-of-hospital and Interhospital wound infection following crotalid envenomation. J
Management of Crotaline Snakebite. Ann Emerg Med Emerg Med 1993;11:583-586. (Prospective, observation-
2001;37(2)168-174 (Review) al; 54 patients)
18. Otten EJ. Venomous Animal Injuries. In: Marx JA, edi- 35. Weed HG, Nonvenomous snakebite in Massachusetts:
tor-in-chief. Rosen’s Emergency Medicine: Concepts prophylactic antibiotics are unnecessary. Ann Emerg
and Clinical Practice, 6th ed. Philadelphia: Mosby Med 1993; 22:220-224. (Prospective observational; 72
Elsevier; 2006. 894-913 (Textbook Chapter) patients)
19. Messmore HL, Jeske WP, Wehrmacher W, el al. 36. Talan DA, Citron D.M, Overturf G.D, et
Antiplatelet Agents: Current drugs and future trends. al. Antibacterial activity of crotalid venoms against
Hematol Oncol Clin N Am 2005;19:87-117. (Review) oral snake flora and other clinical bacteria. J Infect Dis
20. Suncoast Herpetological Society. Venom1 Antivenom 1991;164:195-198. (Basic Science)
Bank. May 2006. Accessed 12 September 2006 at 37. Protherics. “CROFAB® CROTALIDAE POLYVALENT
http://www.kingsnake.com/suncoastherpsociety/SH IMMUNE FAB (OVINE)” Package Insert. 2006.
Sven.htm. (Package Insert)
21. Cheng D. Scorpion Sting. Accessed 25 May 2006 at 38. Quarre JP, Lecomte J, Lauwers D, et al. Allergy to latex
http://www.emedicine.com. (Review) and papain. J Allergy Clin Immunol 1995; 95(4):922.

EBMedicine.net • September 2006 23 Emergency Medicine Practice©


(Case Report) 53. American College of Surgery, Commission on Trauma
39. Baur X, Chen Z, Rozynek P, Düser D, et al. Cross-react- and American College of Emergency Physicians.
ing IgE antibodies recognizing latex allergens, includ- Guidelines for treatment (poster publication). Chicago,
ing Hev b 1, as well as papain. Allergy 1995;50(7):604- IL: American College of Surgery; 1994. (Clinical
609. (Clinical Trial, 58 patients) Consensus Statement)
40. Caravati EM, Copperhead bites and Crotalidae polyva- 54. Sutherland SK, Coulter AR, Harris RD. Rationalisation
lent immune Fab (ovine): Routine use requires evi- of first aid measures for elapid snakebite. Lancet
dence of improved outcomes. Ann Emerg Med 1979;1(8109):183-185. (Expert opinion)
2004;43(2): 207-208. (Expert opinion) 55. Howarth DM, Southee AE, Whyte IM. Lymphatic flow
41. Lavonas EJ, Gerardo CJ, O’Malley G, et al. Initial expe- rates and first aid in simulated peripheral snake or spi-
rience with Crotalidae polyvalent immune Fab (ovine) der envenomation. Med J Aust 1994;161:695-700.
antivenom in the treatment of copperhead snakebite. (Animal study, randomized, controlled)
Ann Emerg Med 2004;43:200-206. (Retrospective chart 56. Bouaziz M, Bahloul M, Hergafi L, et al. Factors associ-
review;32 patients) ated with pulmonary edema in severe scorpion sting
42. Jurkovich GJ, Luterman A, McCullar K, et al. patients—a multivariate analysis of 428 cases. Clin
Complications of Crotalidae antivenin therapy. J Trauma Toxicol (Phila) 2006;44(3):293-300. (Retrospective,
1988;28(7):1032-1037. (Prospective, Cohort; 40 patients) cohort;428 patients)
43. Schmidt JM. Antivenom therapy for snakebites in chil- 57. Bahloul M, Chaari A, Khlaf-Bouaziz N, et al.
dren: is there evidence? Curr Opin Pediatrics Gastrointestinal manifestations in severe scorpion
2005;17(2):234-238. (Review) envenomation. Gastroenterol Clin Biol 2005;29(10):1001-
44. Offerman SR, Bush SP, Moynihan JA, et al. Crotaline 1005. (Retrospective chart review; 951 patients)
Fab antivenom for the treatment of children with rat- 58. Moran NF, Newman WJ, Theakston RD, et al. High
tlesnake envenomation. Pediatrics 2002;110: 968–971. incidence of early anaphylactoid reaction to SAIMR
(Prospective/Retrospective, pediatric cohort; 12 polyvalent snake antivenom. Trans R Soc Trop Med
patients) Hyg. 1998;92(1):69-70. (Prospective case series; 17
45. Dart RC, McNally J. Efficacy, safety, and use of snake patients)
antivenoms in the United States. Ann Emerg Med 59. Isbister GK, Tankel AS, White J, et al. Med J Aus.
2001;37(2):181-188. (Review) 2006;184(8):419-20. (Prospective case series; 14
46. Holstege CP, Wu J, Baer AB. Immediate hypersensitivi- patients)
ty reaction associated with the rapid infusion of 60. LoVecchio F, Welch S, Klemens J, et al. Incidence of
Crotalidae polyvalent immune Fab (ovine). Ann Emerg immediate and delayed hypersensitivity to
Med 2002; 39(6):677-679. (Case report) Centruroides antivenom. Ann Emerg Med
47. Clark RF, McKinney PE, Chase PB. et al. Immediate 1999;34(5):615-619. (Prospective, cohort;116 patients)
and delayed allergic reactions to Crotalidae polyvalent 61. Lo Vecchio F, Klemens J, Roundy EB, et al. Serum sick-
immune Fab (ovine) antivenom. Ann Emerg Med ness following administration of Antivenin
2002;39(6):671-676. (Crotalidae) Polyvalent in 181 cases of presumed rat-
48. Boyer LV, Seifert SA, Clark RF, et al. Recurrent and tlesnake envenomation. Wilderness Environ Med
persistent coagulopathy following pit viper envenoma- 2003;14(4)220-221. (Retrospective;181 patients)
tion. Arch Intern Med 1999;159(7):706-710. 62. Premawardhena AP, deSilva CE, Fonseka MMD, et al.
(Retrospective chart review; 38 patients) Low dose subcutaneous adrenaline to prevent acute
49. Boyer LV, Seifert SA, Cain JS. Recurrence phenomena adverse reactions to antivenom serum in people bitten
after immunoglobulin therapy for snake envenoma- by snakes: randomized, placebo controlled trial. BMJ
tions: part 2. Guidelines for clinical management with 1999;318:1041-1043. (Prospective, randomized, con-
Crotaline Fab antivenom. Ann Emerg Med trolled;105 patients)
2001;37(2):196-201. (Review and clinical guidelines) 63. Nuchpraryoon I, Garner P. Interventions for prevent-
50. Holstege CP, Miller MB, Wermuth M, et al. Crotalid ing reactions to snake antivenom. Cochrane Database
snake envenomation. Crit Care Clin 1997;13(4):889-921. Syst Rev 2000;(2):CD002153. (Systematic review)
(Review Article) 64. LoVecchio F, McBride C. Scorpion envenomations in
51. Stewart RM, Page CP, Schwesinger WH, et al. young children in central Arizona. J Toxicol Clin
Antivenin and fasciotomy/debridement in the treat- Toxicol 2003;41(7):937-40. (Prospective case series; 483
ment of the severe rattlesnake bite. Am J Surg patients)
1989;158:543-547. (Animal Study, randomized, con- 65. Riley BD, LoVecchio F, Pizon AF. Lack of Scorpion
trolled) Antivenom Leads to Increased Pediatric ICU
52. Tanen DA, Danish DC, Grice GA, et al. Fasciotomy Admissions. Ann Emerg Med 2006;47(4):398-399.
worsens the amount of myonecrosis in a porcine (Before-after cohort; about 35,000 patients)
model of Crotaline envenomation. Ann Emerg Med 66. Gupta V. Prazosin: a pharmacological antidote for
2004;44(2):99-104. (Randomized, blinded, controlled scorpion envenomation. J Trop Pediatr 2006;52(2):150-
animal; 20 pigs) 151. (Prospective, before-after, quasi-experimental;36

Emergency Medicine Practice© 24 September 2006 • EBMedice.net


patients) d. Grade III/IV (severe/very severe envenoma-
67. McClain C. 2-year scorpion-antivenin trials successful; tion)
FDA scrutiny next. Accessed 6 July 2006 at
36. If the patient in question three developed no
http://www.azstarnet.com/allheadlines/124591.
(Unpublished, prospective, randomized, controlled;
further symptoms, what is the correct manage-
50 patients) ment plan?
68. Hughes A. Observation of snakebite victims: is twelve
hours still necessary? Emerg Med (Freemantle) a. Mix CroFabTM and administer four vials
2003;15(5-6):511-517. (Two phase case series; 360 according to the treatment algorithm and
patients) then discharge home.
69. Norris R. “Snake Venom Poisoning In The United
b. Mix CroFabTM and administer four vials
States: A Medical Emergency!” Accessed 11 September
according to the treatment algorithm and
2006 at http://www.emed.stanford.education/didac-
tics/snakebites.htm. then admit for the completion of the
70. Gibly R, Williams M, Walter FG et al. Continuous CroFabTM treatment regimen.
intravenous midazolam infusion for Centruroides exili- c. Observe for four hours and discharge if
cauda scorpion envenomation. Ann Emerg Med unchanged.
1999;34(5): 669-670. d. Observe for eight hours and discharge if
unchanged.
CME Questions
37. If the patient in question three developed sig-
33. Which of the following anatomic regions is the
nificant altered mental status, but no other
location for more than 95% of snake enveno-
changes in his presentation, what grade should
mations?
be assigned to his envenomation?
a. Back
a. Grade 0 (nonenvenomation)
b. Extremities
b. Grade I (mild envenomation)
c. Head
c. Grade II (moderate envenomation)
d. Torso
d. Grade III/IV (severe/very severe envenoma-
tion)
34. Which of the following is a true statement?

38. An 8-year-old, 35kg female presents to the


a. 25% of venomous snakebites in the US are
emergency department one hour after an iden-
fatal.
tified cottonmouth/water moccasin bite to the
b. 98% of venomous snakebites in the US are
leg while swimming in a local pond. She has
from Crotalids.
marked pain, severe edema and erythema to
c. Imported snakes account for the majority of
her entire leg, and has an elevated INR and
fatal snakebites in the US.
aPTT. Appropriate management should
d. Up to 20% of the venomous snakebites in the
include which of the following?
US can be attributed to coral snakes.

a. Three vials of CroFabTM, adjusted for weight,


35. A 25-year-old male presents to the ED 60 min-
mixed in 250cc NS, and given according to
utes after a brown snake bit his hand. He says
the treatment protocol.
the snake had a triangular head and a rattle on
b. Six vials of CroFabTM, with no weight adjust-
its tail. Currently, the patient has local
ment, mixed in 250cc NS and given accord-
swelling around 2 fang marks, some mild pain
ing to the treatment protocol.
in his hand, and minimal local swelling. His
c. Pretreatment with epinephrine 0.25mg IM
vitals and labs are normal. What Grade is this
prior to CroFabTM administration.
envenomation?
d. Skin testing to determine sensitivity to
CroFabTM.
a. Grade 0 (nonenvenomation)
b. Grade I (mild envenomation)
39. All of the following patients have received a
c. Grade II (moderate envenomation)

EBMedicine.net • September 2006 25 Emergency Medicine Practice©


dose of four to six vials of CroFabTM. Which of surgery for fasciotomy if greater than
the following has achieved “initial control”? 40mmHg.
d. Measure compartment pressures and admin-
a. Swelling and redness have improved accord- ister an additional four to six vials of
ing to the pen marks, and repeat labs show CroFabTM with mannitol and arm elevation if
an increase in INR from 2.5 to 3.0. pressures are greater than 40mmHg.
b. Swelling and redness have progressed
beyond the pen marks and the repeat labs 42. Which of the following is a true statement?
have returned to baseline normal levels.
c. Swelling and redness have remained the a. Mojave rattlesnake envenomations present in
same according to the pen marks and the a similar fashion to other Crotalid enveno-
repeat labs are improved. mations and can be assessed using the grad-
d. Swelling and redness have remained the ing scale in Table 1.
same according to the pen marks, the repeat b. Coral snake venom is classically considered
labs are improved, and the patient has devel- to be primarily cardiotoxic.
oped confusion. c. Crotalid venom is a complex mixture of pro-
teins and other substances that affect the car-
40. The patient has received six vials of CroFabTM diac, neurologic, hematologic, and muscu-
and has achieved initial control of the enveno- loskeletal systems.
mation. Three hours after his first two vial fol- d. Venom extraction devices are an effective
low-up dose of CroFabTM, his altered mental field treatment for envenomations and
status returns. What is the best course of should be applied by bystanders or EMS
action at this point? providers that have them available in the
field.
a. Administer two vials of CroFabTM and con-
tinue with his scheduled dosing regimen 43. An 18-month-old, 12kg female was stung on the
according to the original times. leg by a scorpion while playing on her porch in
b. Administer two vials of CroFabTM and reset Tuscon, Arizona. Over the past few minutes,
the follow vial timing for additional doses at she has started getting agitated, sweating pro-
6, 12, and 18 hours following the newest fusely, vomiting, and passing watery stools. On
dose. arrival to the ED by EMS, the patient was alert,
c. Administer four to six vials of CroFabTM and agitated, diaphoretic, tachypneic at 48 breaths
reassess in one hour. per minute with oxygen saturation 94% by
d. Wait until the next six hour incremental dose pulse oximetry, and tachycardic at 160 beats per
is due and administer two vials. minute. After starting an IV line, placing oxy-
gen by mask, and instituting continuous moni-
41. A patient who has received one dose of four toring, your best initial course of action is to:
vials of CroFabTM following a severe (Grade III)
envenomation of the hand has developed a. Administer three vials scorpion antivenom
increased pain, pallor, and numbness of his intravenously immediately with a 20ml/kg
hand. Palpation of his forearm reveals a bolus of normal saline.
very tight compartment and he has pain out b. Give 0.12mg epinephrine 1:1000 intramuscu-
of proportion. What is the best course of larly in the thigh and 12.5mg diphenhy-
action? dramine intravenously.
c. Give midazolam intravenously, draw labs,
a. Continue CroFabTM treatment according to and plan for pediatric intensive care unit
timed protocol, elevate his arm, and reexam- admission.
ine in one hour. d. Give metoprolol 2.5mg intravenously every
b. Immediate fasciotomy to relieve compart- five minutes for three doses to establish beta-
ment sydrome. receptor blockade, but hold for heart rate
c. Measure compartment pressures and consult less than 60 beats per minute.

Emergency Medicine Practice© 26 September 2006 • EBMedice.net


44. The patient is a 39-year-old man who was bitten the patient is anxious, tachypneic at 32 breaths
on the hand while clearing brush at his Florida per minute, with faint wheezes bilaterally.
home by a red, yellow, and black snake, After establishing IV access, placing oxygen by
thought to be a coral snake. Injury occurred 30 nasal cannula, and instituting continuous car-
minutes before ED arrival and the patient has diac monitoring, what is the best initial course
no symptoms, no pain, no paresthesias, and no of action?
neurological deficits. What is the best course
of action? a. Give 0.3mg epinephrine 1:1000 intramuscu-
larly in the thigh and 50mg diphenhy-
a. Observe for 12 hours and, if no symptoms, dramine intravenously.
discharge home. b. Give three more vials of coral snake antiven-
b. Obtain CBC, CMP, PT/PTT and fibrinogen at om intravenously for progression of symp-
baseline and after four hours while monitor- toms.
ing for symptoms. c. Perform neuro checks every hour to assess
c. Immediately give coral snake antivenom. for progression of symptoms.
d. Perform neuro checks every hour and give d. Perform rapid sequence intubation for
antivenom when and if symptoms appear. impending respiratory failure.

45. The patient is a 26-year-old man who was clean- 47. The patient is a 44-year-old man who received
ing up in the trailer of a circus snake handler polyvalent, horse serum-derived antivenom for
and decided to “mess” with the snakes. A bites from a Gaboon viper while working at
monocle cobra bit him at least twice and the zoo two weeks ago. He presents to the ED
maybe three times on the right hand and fore- for evaluation of an itchy, red rash on trunk
arm about two hours ago. He tried to conceal and arms that has been increasing for the past
his injury from his boss, but started to have three days. What is the best treatment plan for
severe pain, muscle twitching, and difficulty this patient?
swallowing so admitted the injury and was
brought into the ED. The snake handler brings a. Obtain additional Gaboon viper antivenom
ten vials of polyvalent cobra antivenom with to give additional treatment for unresolved
the patient. What is the best initial course of envenomation.
action? b. Give 0.3mg epinephrine 1:1000 intramuscu-
larly in the thigh and 50mg diphenhy-
a. Call the poison control and the regional zoo dramine intravenously.
for guidance in managing this patient. c. Obtain CBC, CMP, PT/PTT and fibrinogen at
b. Give midazolam 4mg intravenously, draw baseline and after four hours while monitor-
labs, and plan for ICU admission. ing for symptoms.
c. Give 0.25mg epinephrine subcutaneously to d. Give prednisone 60mg and diphenhy-
prevent immediate hypersensitivity reaction dramine 50mg PO in the ED and prescribe a
and three vials of cobra venom intravenous- steroid taper for ten days and prn diphenhy-
ly. dramine.
d. Obtain CBC, CMP, PT/PTT, and fibrinogen
at baseline and perform type and cross for 48. The patient is a 36-year-old collector of ven-
packed red blood cells and fresh frozen plas- omous snakes who was bitten by one of his
ma. black mamba (Dendroaspis polylepis) snakes
about one hour ago. He has no clear symptoms,
46. The patient is a 52-year-old woman who was but says he feels funny and anxious. His vital
bitten by a coral snake two hours ago and signs are stable and he is in no distress. He
given three vials of coral snake antivenom reports that he has antivenom for almost all of
about ten minutes ago. She complains of an his snakes, but he has never been able to obtain
intensely itchy, blotchy rash all over the body antivenom for this snake species. After estab-
and face and swelling of the lips. On exam, lishing IV access, placing oxygen by nasal can-

EBMedicine.net • September 2006 27 Emergency Medicine Practice©


nula, and instituting continuous cardiac moni- Committees of the American Heart emergency cardiac care. Emergency
Association and representatives Cardiac Care Committee and
toring, what is the best initial course of action? from the resuscitation councils of Subcommittees, American Heart
ILCOR: How to Develop Evidence- Association. Part IX. Ensuring effec-
Based Guidelines for Emergency tiveness of community-wide emer-
a. Call your local poison center or national poi- Cardiac Care: Quality of Evidence gency cardiac care. JAMA
and Classes of Recommendations; 1992;268(16):2289-2295.
son center hotline 800-222-1222 for help also: Anonymous. Guidelines for
cardiopulmonary resuscitation and
obtaining antivenom.
b. Give coral snake antivenom three vials intra-
venously to capitalize on cross-reactivity of
the species.
Physician CME Information
c. Give 0.3mg epinephrine 1:1000 intramuscu- Credit Designation: The Mount Sinai School of Medicine designates this
educational activity for a maximum of 48 AMA PRA Category 1
larly in the thigh and 50mg diphenhy- Credit(s)TM per year. Physicians should only claim credit commensurate
with the extent of their participation in the activity.
dramine intravenously.
Credit may be obtained by reading each issue and completing the printed
d. Obtain CBC, CMP, PT/PTT, and fibrinogen post-tests administered in December and June or online single-issue
post-tests administered at EBMedicine.net.
at baseline and after four hours while moni-
Target Audience: This enduring material is designed for emergency medi-
toring for symptoms. cine physicians.
Needs Assessment: The need for this educational activity was determined
by a survey of medical staff, including the editorial board of this publica-
tion; review of morbidity and mortality data from the CDC, AHA, NCHS,
and ACEP; and evaluation of prior activities for emergency physicians.
Date of Original Release: This issue of Emergency Medicine Practice was
Coming in Future Issues: published September 12, 2006. This activity is eligible for CME credit
through September 12, 2009. The latest review of this material was
Acutely Decompensated Heart Failure September 1, 2006.
Weakness Discussion of Investigational Information: As part of the newsletter, fac-
Complications In Pregnancy ulty may be presenting investigational information about pharmaceutical
products that is outside Food and Drug Administration approved labeling.
Information presented as part of this activity is intended solely as contin-
uing medical education and is not intended to promote off-label use of
Class Of Evidence Definitions any pharmaceutical product. Disclosure of Off-Label Usage: This issue of
Emergency Medicine Practice discusses no off-label use of any pharma-
Each action in the clinical pathways section of Emergency Medicine ceutical product.
Practice receives a score based on the following definitions. Faculty Disclosure: It is the policy of Mount Sinai School of Medicine to
ensure objectivity, balance, independence, transparency, and scientific
Class I Class III rigor in all CME-sponsored educational activities. All faculty participating
• Always acceptable, safe • May be acceptable in the planning or implementation of a sponsored activity are expected to
• Definitely useful • Possibly useful disclose to the audience any relevant financial relationships and to assist
• Proven in both efficacy and • Considered optional or alternative in resolving any conflict of interest that may arise from the relationship.
effectiveness treatments Presenters must also make a meaningful disclosure to the audience of
their discussions of unlabeled or unapproved drugs or devices.
Level of Evidence: Level of Evidence: In compliance with all ACCME Essentials, Standards, and Guidelines, all
• One or more large prospective • Generally lower or intermediate faculty for this CME activity were asked to complete a full disclosure
studies are present (with rare levels of evidence statement. The information received is as follows: Dr. Costello, Dr. Heins,
exceptions) • Case series, animal studies, con- Dr. Zirkin, Dr. Lovecchio, Dr. Barish, and Dr. Stewart report no significant
• High-quality meta-analyses sensus panels financial interest or other relationship with the manufacturer(s) of any
• Study results consistently positive • Occasionally positive results commercial product(s) discussed in this educational presentation.
and compelling
For further information, please see The Mount Sinai School of Medicine
Indeterminate
website at www.mssm.edu/cme.
Class II • Continuing area of research
• Safe, acceptable • No recommendations until further ACEP Accreditation: Emergency Medicine Practice is approved by the
• Probably useful research American College of Emergency Physicians for 48 hours of ACEP
Category 1 credit per annual subscription.
Level of Evidence: Level of Evidence: AAFP Accreditation: Emergency Medicine Practice has been reviewed
• Generally higher levels of evidence • Evidence not available and is acceptable for up to 48 Prescribed credits per year by the
• Non-randomized or retrospective • Higher studies in progress American Academy of Family Physicians. AAFP Accreditation begins
studies: historic, cohort, or case- • Results inconsistent, contradictory August 1, 2006. Term of approval is for two years from this date. Each
control studies • Results not compelling issue is approved for 4 Prescribed credits. Credits may be claimed for
• Less robust RCTs two years from the date of this issue.
• Results consistently positive Significantly modified from: The
AOA Accreditation: Emergency Medicine Practice has been approved for
Emergency Cardiovascular Care
48 Category 2B credit hours per year by the American Osteopathic
Association.

Emergency Medicine Practice is not affiliated with any pharmaceutical firm or medical device manufacturer.

CEO: Robert Williford President and Publisher: Stephanie Williford

Direct all editorial or subscription-related questions to EB Medicine: 1-800-249-5770 • Fax: 1-770-500-1316 • Non-U.S. subscribers, call: 1-678-366-7933
EB Practice, LLC • 305 Windlake Court • Alpharetta, GA 30022
E-mail: emp@empractice.net • Web Site: EBMedicine.net
Emergency Medicine Practice (ISSN Print: 1524-1971, ISSN Online: 1559-3908) is published monthly (12 times per year) by EB Practice, LLC, 305 Windlake Court, Alpharetta, GA 30022. Opinions
expressed are not necessarily those of this publication. Mention of products or services does not constitute endorsement. This publication is intended as a general guide and is intended to supple-
ment, rather than substitute, professional judgment. It covers a highly technical and complex subject and should not be used for making specific medical decisions. The materials contained herein
are not intended to establish policy, procedure, or standard of care. Emergency Medicine Practice is a trademark of EB Practice, LLC. Copyright © 2006 EB Practice, LLC. All rights reserved. No
part of this publication may be reproduced in any format without written consent of EB Practice, LLC. Subscription price: $299, U.S. funds. (Call for international shipping prices.)

Emergency Medicine Practice© 28 September 2006 • EBMedice.net

You might also like